Ecuación de Dirac en Relatividad General

La ecuación de Dirac para los fermiones sin masa en un espacio de tiempo curvo es γ a mi a m D m Ψ = 0 , dónde mi a m son las tétradas. Tengo que demostrar que los espinores de Dirac obedecen a la siguiente ecuación:

( D m D m + 1 4 R ) Ψ = 0 ( 1 )

dónde R es el escalar de Ricci.

eso ya lo se [ D m , D v ] A ρ = R m v ρ σ A σ , pero un punto clave es saber qué [ D m , D v ] Ψ es.

( D m Ψ = m Ψ + A m a b Σ a b es la derivada covariante del campo de espinor y Σ a b los generadores de Lorentz que involucran matrices gamma).

La forma correcta de resolverlo es actuar con D m desde la izquierda en su ecuación Dirac GR nuevamente. Una vez que lo haga, debe darse cuenta de que la nueva derivada covariante también actúa sobre las tétradas, lo que significa que efectivamente diferencia la métrica y, en última instancia, produce el término escalar de Ricci. Sin embargo, debes darte cuenta de que el adjetivo "covariante" significa que tiene dos nuevos términos de conexión, uno de la métrica del espacio-tiempo curvo y otro del potencial electromagnético. Este último no es necesario en absoluto para la derivación deseada: el campo de indicador se mantiene en todas partes como parte de D m .
Relacionado está physics.stackexchange.com/questions/51269/… ¿Son realmente distintos? De lo contrario, podría considerar pedirles a los mods que los fusionen.
Gracias por el comentario, pero D m mi a v no es cero?
¿No son las tétradas covariantemente constantes siempre que incluya tanto los símbolos de Christoffel como la conexión de espín?
¡Sí lo son! ¿Puedes escribir explícitamente lo que quieres decir para D m mi a v y cómo esto puede resolver el problema... Ahora estoy un poco confundido acerca de la acción adecuada de D m en tétradas.
Diría que el derivado cov de la tétrada es algo así como m mi v a Γ m v σ mi σ a + ω m a b mi v b eran ω m a b es la conexión de espín. (Griego es espacio-tiempo y latín es etiqueta de tétrada, también mantendré cualquier cosa de calibre EM fuera de aquí). (PD: ¡no pretendo haber resuelto tu problema!)

Respuestas (1)

denotar por γ a las matrices gamma del espacio de Minkowski con respecto a la tétrada de Lorentz { mi a } y derivada covariante D a , entonces las gammas son covariantemente constantes.

Comience con la ecuación de Dirac sin masa

γ b D b Ψ = 0

Actúa de nuevo con el operador de Dirac

γ a D a γ b D b Ψ = 0
Entonces, desde D aniquila γ
γ a γ b D a D b Ψ = 0
entonces
1 2 { γ a , γ b } D a D b Ψ + 1 2 γ a γ b [ D a , D b ] Ψ = 0     ( 1 )
Pero
{ γ a , γ b } = 2 η a b
y
[ D a , D b ] Ψ = R a b Ψ
Dónde R a b es la curvatura de espín (antisimétrica en a y b). R a b satisface la identidad
γ b R a b = R a b γ b = 1 2 γ b R a b
dónde R a b es el tensor de Ricci (en la tétrada de Lorentz). entonces (1) se convierte en
[ D a D a + 1 4 γ a γ b R a b ] Ψ = 0
es decir
[ D a D a 1 4 R ] Ψ = 0

Su respuesta ha inspirado una pregunta de seguimiento. .